www.vorkurse.de
Ein Projekt von vorhilfe.de
Die Online-Kurse der Vorhilfe

E-Learning leicht gemacht.
Hallo Gast!einloggen | registrieren ]
Startseite · Mitglieder · Teams · Forum · Wissen · Kurse · Impressum
Forenbaum
^ Forenbaum
Status Mathe-Vorkurse
  Status Organisatorisches
  Status Schule
    Status Wiederholung Algebra
    Status Einführung Analysis
    Status Einführung Analytisc
    Status VK 21: Mathematik 6.
    Status VK 37: Kurvendiskussionen
    Status VK Abivorbereitungen
  Status Universität
    Status Lerngruppe LinAlg
    Status VK 13 Analysis I FH
    Status Algebra 2006
    Status VK 22: Algebra 2007
    Status GruMiHH 06
    Status VK 58: Algebra 1
    Status VK 59: Lineare Algebra
    Status VK 60: Analysis
    Status Wahrscheinlichkeitst

Gezeigt werden alle Foren bis zur Tiefe 2

Navigation
 Startseite...
 Neuerdings beta neu
 Forum...
 vorwissen...
 vorkurse...
 Werkzeuge...
 Nachhilfevermittlung beta...
 Online-Spiele beta
 Suchen
 Verein...
 Impressum
Das Projekt
Server und Internetanbindung werden durch Spenden finanziert.
Organisiert wird das Projekt von unserem Koordinatorenteam.
Hunderte Mitglieder helfen ehrenamtlich in unseren moderierten Foren.
Anbieter der Seite ist der gemeinnützige Verein "Vorhilfe.de e.V.".
Partnerseiten
Weitere Fächer:

Open Source FunktionenplotterFunkyPlot: Kostenloser und quelloffener Funktionenplotter für Linux und andere Betriebssysteme
Forum "Uni-Stochastik" - Varianz der Stichprobenvarianz
Varianz der Stichprobenvarianz < Stochastik < Hochschule < Mathe < Vorhilfe
Ansicht: [ geschachtelt ] | ^ Forum "Uni-Stochastik"  | ^^ Alle Foren  | ^ Forenbaum  | Materialien

Varianz der Stichprobenvarianz: Frage (beantwortet)
Status: (Frage) beantwortet Status 
Datum: 18:18 Mi 14.11.2007
Autor: chimneytop

Aufgabe
[mm] X_i\sim P(\lambda) [/mm] für alle i. Berechnen Sie die Varianz des Schätzers [mm] S_n^2 [/mm] für n=2.

[mm] S_n^2=\bruch{1}{n}\summe_{i=1}^n(X_i-\overline{X_n})^2 [/mm]
[mm] \overline{X_n}=\bruch{1}{n}\summe_{i=1}^n(X_i) [/mm]

Ich erhalte

[mm] S_2^2=\bruch{1}{4}(X_1-X_2)^2 [/mm]

Was ist der einfachste Weg davon die Varianz auszurechnen?

Ich hab versucht umzuformen:

[mm] S_2^2=\bruch{1}{2}(Var(X_1^2)-Var(X_1*X_2)) [/mm] (da die Varianzen von [mm] X_1^2 [/mm] und [mm] X_2^2 [/mm] gleich sind.

Den ersten Teil hab ich mal mit Mathematica ausgerechnet (würd notfalls händisch auch gehn): [mm] \lambda [/mm] + 6 [mm] \lambda^2 [/mm] + 4 [mm] \lambda^3. [/mm]

Beim zweiten müsste ich wieder die gemeinsame Verteilung zweier Poisson-Verteilungen kennen und davon die Varianz bestimmen. Das kommt mir alles sehr komisch vor.

Ist das der richtige Weg oder gehts auch einfacher?

        
Bezug
Varianz der Stichprobenvarianz: Antwort
Status: (Antwort) fertig Status 
Datum: 20:37 Mi 14.11.2007
Autor: luis52


>  Ich erhalte
>  
> [mm]S_2^2=\bruch{1}{4}(X_1-X_2)^2[/mm]
>  

[ok]

> Was ist der einfachste Weg davon die Varianz auszurechnen?
>  
> Ich hab versucht umzuformen:
>  
> [mm]S_2^2=\bruch{1}{2}(Var(X_1^2)-Var(X_1*X_2))[/mm] (da die
> Varianzen von [mm]X_1^2[/mm] und [mm]X_2^2[/mm] gleich sind.

Was ist denn das fuer eine komische Formel? Kenne ich nicht.

Du kommst nicht auf die Beine ohne die Annahme, dass [mm] $X_1,X_2$ [/mm] unabhaengig sind.
Kennst du die alte Bauernregel [mm] $\mbox{Var}[U]=\mbox{E}[U^2]-\mbox{E}^2[U]$ [/mm] fuer eine Zufallsvariable $U$?


lg Luis

Bezug
                
Bezug
Varianz der Stichprobenvarianz: Ja, aber ;)
Status: (Korrektur) richtig (detailiert geprüft) Status 
Datum: 22:10 Mi 14.11.2007
Autor: chimneytop

[mm] S_2^2=\bruch{1}{4}(X_1+X_2)^2 [/mm]

Ausmultiplizieren ergibt

[mm] \bruch{1}{4}(X_1^2+2*X_1*X_2+X_2^2) [/mm]

Davon die Varianz ist die Formel in vorigem Posting (ich hab versehentlich [mm] S_2^2 [/mm] statt [mm] Var(S_2^2) [/mm] geschrieben. Dann hab ich nach der Formel [mm] Var[X]=E[X^2]+E[X]^2 [/mm] den ersten Term ausgerechnet. Beim Teil Var(X1*X2) häng ich allerdings.

Bezug
        
Bezug
Varianz der Stichprobenvarianz: Antwort
Status: (Antwort) fertig Status 
Datum: 22:37 Mi 14.11.2007
Autor: luis52


> $ [mm] S_2^2=\bruch{1}{4}(X_1+X_2)^2 [/mm] $

Wieso denn das auf einmal? Ich denke

$ [mm] S_2^2=\bruch{1}{4}(X_1-X_2)^2 [/mm] $

>Ausmultiplizieren ergibt


>$ [mm] \bruch{1}{4}(X_1^2+2\cdot{}X_1\cdot{}X_2+X_2^2) [/mm] $

und folglich $ [mm] \bruch{1}{4}(X_1^2-2\cdot{}X_1\cdot{}X_2+X_2^2) [/mm] $

Nenne [mm] $U=\bruch{1}{4}(X_1^2-2\cdot{}X_1\cdot{}X_2+X_2^2) [/mm] $

und berechne [mm] $\mbox{E}[U^2)]-\mbox{E}^2[U]$. [/mm] Dann brauchst du auch
nicht mit der ominoesen [mm] $\mbox{Var}[X_1X_2]$ [/mm] zu rechnen.


lg luis


Bezug
                
Bezug
Varianz der Stichprobenvarianz: Korrekturmitteilung
Status: (Korrektur) oberflächlich richtig Status 
Datum: 23:27 Mi 14.11.2007
Autor: chimneytop

Ok, Denkfehler, bei Unabhängigkeit ist natürlich E(X*Y)=E(X)*E(Y).

Dann erhalte ich ziemlich schnell:

[mm] E(S_2^2)=\bruch{1}{4}*E(X_1-X_2)^2 [/mm]
[mm] =\bruch{1}{4}*E(X_1-X_2)*E(X_1-X_2) [/mm]
[mm] =\bruch{1}{4}*(E(X_1)-E(X_2))*(E(X_1)-E(X_2)) [/mm]
[mm] =\bruch{1}{4}*(\lambda-\lambda)*(\lambda-\lambda) [/mm]
=0.

Analog für [mm] E(S_2^2)^2=\bruch{1}{16}*E(X_1-X_2)^4=0. [/mm]

Folglich wäre [mm] Var[S_2^2]=0. [/mm]

Ist das richtig?

Gruß

Bezug
        
Bezug
Varianz der Stichprobenvarianz: Antwort
Status: (Antwort) fertig Status 
Datum: 07:31 Do 15.11.2007
Autor: luis52


> $ [mm] E(S_2^2)=\bruch{1}{4}\cdot{}E(X_1-X_2)^2 [/mm] $
> $ [mm] =\bruch{1}{4}\cdot{}E(X_1-X_2)\cdot{}E(X_1-X_2) [/mm] $
> $ [mm] =\bruch{1}{4}\cdot{}(E(X_1)-E(X_2))\cdot{}(E(X_1)-E(X_2)) [/mm] $

?$ [mm] =\bruch{1}{4}\cdot{}(\lambda-\lambda)\cdot{}(\lambda-\lambda) [/mm] $

> =0.

> Analog für $ [mm] E(S_2^2)^2=\bruch{1}{16}\cdot{}E(X_1-X_2)^4=0. [/mm] $

> Folglich wäre $ [mm] Var[S_2^2]=0. [/mm] $

> Ist das richtig?

Offensichtlich nicht, das [mm] $S^2>0$. [/mm] Du kannst nicht unterstellen

> $ [mm] E(S_2^2)=\bruch{1}{4}\cdot{}E(X_1-X_2)^2 [/mm] $
> $ [mm] =\bruch{1}{4}\cdot{}E(X_1-X_2)\cdot{}E(X_1-X_2) [/mm] $


etwa indem [mm] $X_1-X_2$ [/mm] und [mm] $X_1-X_2$ [/mm] unabhaengig sind. Multipliziere stattdessen
[mm] $(X_1-X_2)^2 [/mm] $ aus und bilde dann den Erwartungswert. Analog fuer
[mm] $(X_1-X_2)^4$. [/mm]


lg Luis

PS: Koenntest du bitte deine Folgebeitraege als Mitteilungen oder als
Fragen formulieren (nicht als Korrekturmitteilung). Das wuerde die Korrespondenz
erheblich erleichtern. Danke.

Bezug
Ansicht: [ geschachtelt ] | ^ Forum "Uni-Stochastik"  | ^^ Alle Foren  | ^ Forenbaum  | Materialien


^ Seitenanfang ^
www.vorkurse.de
[ Startseite | Mitglieder | Teams | Forum | Wissen | Kurse | Impressum ]